Let u = [3 8 1 −4], v = [1 3 3 4], and let W be the subspace of R4 spanned by u and v. Find a basis of W⊥, the orthogonal complement of W in R4.

Algebra & Trigonometry with Analytic Geometry
13th Edition
ISBN:9781133382119
Author:Swokowski
Publisher:Swokowski
Chapter8: Applications Of Trigonometry
Section8.4: The Dot Product
Problem 19E
icon
Related questions
Question

Let u = [3 8 1 −4], v = [1 3 3 4], and let W be the subspace of R4 spanned by u and v. Find a basis of W⊥, the orthogonal complement of W in R4.

Expert Solution
trending now

Trending now

This is a popular solution!

steps

Step by step

Solved in 4 steps with 3 images

Blurred answer